Download as docx, pdf, or txt
Download as docx, pdf, or txt
You are on page 1of 45

CRIMINAL LAW OUTLINE

CHAPTER ONE- INTRODUCTION: SETTING THE STAGE


A. Nature, Sources, and Limits of the Criminal Law
The method of Criminal Law operates by a series of commands, formulated in general
terms, telling people what they must or must not do.
o Two scholars have defined a crime as "any social harm defined and made
punishable by law."
o The Model Penal Code has greatly influenced criminal law reform.
B. Criminal Law in a Procedural Context: Pre-Trial
According to the NCVS, 18.7 million people 12 or older, were victims of violent or
property crimes in 2010.
However, only about 50% of violent victimizations and 40% or property crimes were
reported to Law Enforcement Agencies. - U.S.Dept. of Justice, Bureau Statistics.
In many states and in the Federal System, the accused may not be brought to trial
unless they are indicted by a grand jury.
C. Criminal law in a Procedural Context: Trial by Jury
The Sixth Amendment says that "in all criminal prosecutions, the accused shall enjoy
the right to a speedy and public trial, by an impartial jury."
Burden of Proof for a Civil Case- More likely than not it's true.
Burden of Proof for a Criminal Case- Beyond a reasonable doubt (firmly convinced)
D. Proof of Guilt at Trial
1. Proof Beyond a Reasonable Doubt
o Burden of Proof required in a Criminal case.
2. Enforcing the Presumption of Innocence
o Owens v. State- Found asleep in a private drive, with beer in between legs, and
car running. Attrial,Owenarguedtherewasinsufficientevidencetoprovehehad
beendrivingdrunkonapublichighway.Thetrialcourtfoundhimguilty.
Issue: Is circumstantial evidence alone legally sufficient to prove guilt at
trial? Yes.
Rule: A conviction may be based on circumstantial evidence alone if the
circumstances are such that they are inconsistent with any reasonable
theory or argument there may be defending their innocence.
Holding: Yes. In the absence of direct evidence, the jury must make
reasonable inferences from circumstantial evidence to determine whether a
criminal defendant is guilty A jury should not rely solely on these
inferences to convict a defendant unless the circumstances are
incompatible with any reasonable theory of innocence

Presumption of Innocence: The relevant inquiry is whether, after viewing the evidence
in the light most favorable to the prosecution, any rational trier of fact (jury) could have
found the essential elements of the crime proven beyond a reasonable doubt.

PresumptionofInnocenceonAppeal
o Defendant files an appeal based on "insufficiency of evidence" (failed to
overcome the presumption of innocence). The jury is in better position to
resolving conflicting factual claims, so how should an appellate court resolve such
a claim?
E. Jury Nullification
1. JuryNullification A sanctioned doctrine of trial proceedings wherein members of a jury
disregard either the evidence presented or the instructions of the judge in order to reach a
verdict based upon their own conscious.
Powervs.Right

Juries don't have the RIGHT to nullification because they don't have the right to ignore or
depict the law, but ultimately, they have the POWER to resist government and the law if
they feel it necessary.

State v. Ragland- Raglandwaschargedwitharmedrobberyandpossessionofaweaponbya


convictedfelon.Attheendofthetrial,thejudgeinstructedthejurythatitmustfindRagland
guiltyoftheoffenseifthejuryfoundRaglandwascarryingaweaponatthetimeoftherobbery.
Hearguedthatjurysshouldbeinformedofnullificationpowers&thatmustwasinconsistent
withthatpower.
o Issue: Is jury nullification an essential attribute of a criminal defendants right to trial by
jury? No.
o Rule: The power of a jury to nullify the law is not essential to a criminal defendants right to
trial by jury.
2.
WhatdidthedraftersoftheConstitutionsayaboutJuryNullification?
o They wanted them to have the power to do so in the end, so there wasn't issue
over right vs. power or too much governmental power. They wanted them to be
protected from GOVERNMENTAL OPPRESSION.
CHAPTER TWO- PRINCIPLES OF PUNISHMENT
B. The Penal Theories in Action
2. How much (and What) Punishment should be Imposed?
o People v. Superior CT (Du)-Duwasworkingatherliquorstore,whichhadbeen
knownforrecentviolenceandattacksontheirsonbygangmembers.Latashaput
ajuiceinherbagsoDuwaspullingonhersweatertotryandgetit,andLatasha
smackedhertwice.Duthenthrewastoolather,andwhenshemissed,sheshot
andkilledher.Thegunhadbeenalteredpriortothekillingw/oDuknowing.

Issue: Whether the sentence for the commission of a crime with a deadly
weapon should be reduced to probation.
Ruling: A prison sentence for a crime involving a deadly weapon should
not be reduced to probation except in unusual cases where the interests of
justice would best be served
Holding: Since the gun was altered, Du has no record and isnt a threat to
harm others in the future, she was only there because her son had been
attacked and he wanted to be off, and because she was believed to do this
because of the prior circumstances, she got probation.

o The judge must include these things when deciding sentencing: The protection of society,
the need to punish the defendant, and the need to deter further crime, among other objectives.
C. Proportionality of Punishment
2. Constitutional Principles
o Ewing v. California- Ewingwasarrestedforstealinggolfclubsworth$1,200.
Ewinghadpriorconvictions,includingthreeburglariesandarobbery.Hewas
chargedwithfelonygrandtheft,andtheprosecutorinvokedthethreestrikeslaw.
CertiorariwasgrantedtoseeifthissentenceviolatedtheEighthAmendment.
CaliforniasThreeStrikesandYoureOutLaw,defendantswithmore
thantwoviolentorseriousfeloniesaresentencedtoanindeterminate
termoflifeimprisonment.
Issue:Does sentencing a repeat felon to 25 years imprisonment under a
states three strikes law violate the Eighth Amendment prohibition of cruel
and unusual punishment?
Rule: Sentencing a repeat felon to 25 years imprisonment under a states
three strikes law does NOT violate the Eighth Amendment prohibition of
cruel and unusual punishment.
Holding: They said that there were 3 things used to determine if a sentence is
disproportionate enough to violate the 8th.
o 1.) the gravity of the offense and the harshness of the penalty.
o 2.) the sentences imposed on other criminals in same jurisdiction
o 3.) the sentences imposed for commission of same crime in other jurisdictions.
EighthAmendment:Thisamendmentprohibitscruelandunusualpunishments,
(prohibitssentencesthataredisproportionatetothecrime)andalsomentionsexcessive
finesandbail.
o Gravity of an offense not only deals with the current crime at hand, but how it
has been dealt with in the past in the same and in different jurisdictions.

Chapter Three- Modern Role of Criminal Statutes


A. Principle of Legality
1. The Requirement of Previously Defined Conduct

o Keeler v. Superior CT- Shortlyafterobtainingadivorcefromhiswife,Keeler


interceptedheronamountainroad,observedthatshewasvisiblypregnantby
anotherman,andshovedhiskneeintoherabdomenandstruckher.Thereafter,
theviablefetuswasdeliveredstillborn,itsheadfractured.Keelerwascharged
withmurder.
o Issue: Is a viable fetus a human being under the law to which one may be
charged with murder?
o Rule: Proof the child was born alive is necessary to support an indictment for
murder. (Born-Alive Rule)
o Holding: Since the statute in Penal code 187 doesnt deem the fetus as a human
he cant be charged with murder. (Unlawful killing of a human being.

PRINCIPLE OF LEGALITY (COMMON LAW): A person may not be convicted and


punished unless the conduct was defined by criminal statute when committed.
o Three Interrelated Principles (D.U.I.)
1. Statutes should be understandable in order to provide fair notice of
proscribed conduct. (5th and 14th Amendment).
2. Statute should be crafted so they do not provide police, prosecutors,
judges, and juries opportunity to act in a discriminatory or arbitrary
manner.
3. Judicial Interpretation of ambiguous statutes must be construed in favor
of defendant. (Lenity Doctrine).
Ex Post Facto Law: Laws adopted after an act is committed making it illegal although it
was legal when done, or increases the penalty for a crime after it is committed.
How to Distinguish between a Due Process and Ex Post Facto clause violation?
o Due Process Violation- Judicial act (Du=Ju)
o Ex Post Facto violation- Legislature act
The First essential of due process is fair warning of the act, which is made punishable as
a crime.
**When a new penal statute is applied retroactively to make punishable an act, which
was not criminal at the time it was performed, the defendant has been given no advance
noticed consistent with due process.
2. The Values of Statutory Clarity
o City of Chicago v. Morales- In1992,theChicagoCityCouncilenactedthe
GangCongregationOrdinance,whichprohibitedcriminalstreetgangmembers
fromloiteringwithoneanotherinanypublicplace.Moraleswasaccusedof
violatingthelaw,butthetrialcourtthatheldthelawtobeunconstitutionally
vaguedismissedthecharges.
o Issue: Does this ordinance violate the Due Process Clause of the 14th amendment
if it is so vague that it fails to provide sufficient notice to citizens regarding the
type of conduct prohibited and fails to provide adequate guidelines for law
enforcement officers? YES. (Rule stated)
4

A law may be unconstitutionally vague for two reasons.


o First, it may fail to provide the kind of notice that will allow an ordinary person
to understand what conduct is prohibited.
o Second, it may authorize arbitrary enforcement. A law fails to meet the Due
Process requirements of the Fourteenth Amendment if it is so vague that it leaves
the public uncertain as to the conduct it prohibits.
o How did the ordinance bestow police with authority to act arbitrarily or in a
discriminatory manner?
The ordinance is too vague, so they are accusing and participating in innocent
conduct, because they over accuse due to the vagueness and uncertainty of law
enforcement.

B. Statutory Interpretation
Muscarello v. U.S- Congressenactedafederallawcriminalizingtheconductofonewho
usesorcarriesafirearmwhiletraffickingdrugs.Muscarello(D)wasconvictedof
carryingafirearminthelockedglovecompartmentofhisvehiclewhileinvolvedina
drugtraffickingcrime.HeargueditonlyappliedtocarryingitONyou,NOTinthecar.
o Issue: Is the statutory phrase carries a firearm necessarily limited to the
possession a firearm on the defendants person and not also applicable to
possession of a firearm in a vehicle? NO.
o Rule: When interpreting a statutory term, courts should assume the primary
meaning of the phrase governs UNLESS the legislature indicates otherwise.
o Holding: No. Courts must interpret ambiguous statutory terms according to
legislative intent. In doing so, courts may consider the ordinary definition of the
term, any special definitions, the purpose of the statute, and the legislative history.
Congress intended for the provision to apply to the carrying of firearms in a
vehicle to a drug trafficking crime, as Muscarello did. Hes guilty.
What happens when there are multiple meanings to a word, Ex: Carries
o After construing congressional intent, and evaluating all meanings of the word, if
unsure, they will rule in favor of the Defendant.
HYPOTHETICAL:Astateforbidsallraunchybumperstickersoncarsdriveninpublic.
Bobbyisarrestedforviolatingthestatutewhenheiscaughtdrivingdownapublichighwaywith
abumperstickerthatdepictsanudemanandwoman.Shouldhebeconvictedunderthestatute?
Why?
No, because raunchy is too vague. Some people gravitate to the assumption that
those images are raunchy, but the response to that is that there are many things that
we as a society engage in, that can be considered raunchy to some, but not others.
Statute would have to be defined, and specific conduct was banned, would be the only
way to move forward with this conviction.
5

CHAPTER FOUR- ACTUS REUS


A.VoluntaryAct
3. Martin v. State- Martinwasarrestedathishomebypoliceofficersandplacedinapolice

vehiclewherehewasseparatelychargedforbeingdrunkandusingloudandprofanelanguageon
apublichighway.Hewasconvictedunderastatestatutewhichheldthatanypersonwho,while
intoxicatedordrunk,appearedinanypublicplacewhereoneormorepersonsarepresent,and
manifestsadrunkenconditionbyorloudandprofanediscourse,shall,onconviction,be
fined.
o Issue: Can criminal liability be imposed only when the unlawful conduct is
committed voluntarily? YES.
o Rule: Criminal liability may ONLY be imposed when the unlawful conduct is
committed voluntarily.
o Holding: Yes.Undertheplainlanguageofthestatute,oneconvictedofbeing
drunkinapublicplacemusthavevoluntarilyplacedhimselfthere.Iftheaccused
istakeninvoluntarilyandforciblycarriedtoapublicplacebyanarrestingofficer,
achargeofbeingdrunkinapublicplacecannotstand.(ConvictionReversed)

State v. Utter- Killed his son after heavy drinking. Blamed it on Irresistible Impulse
o Issue: Should juries in a murder trial consider evidence on the defense of
conditioned response that tends to demonstrate whether the defendant committed
the requisite Actus Reus? YES.
o Rule: Substantial evidence on the defense of conditioned response tending to
demonstrate whether the defendant committed the requisite Actus Reus should be
presented to the jury in a murder trial.
o Holding: Yes. To find a defendant guilty of murder, a jury must find both the
requisite Actus Reus (physical) and Mens Rea (mental). But Actus Reus also has
a mental component in that the murder defendant must have voluntarily willed the
act to occur. For this reason, an involuntary muscle spasm that results in the
death of another person would not qualify as the Actus Reus for murder. The
act must be committed voluntarily and consciously.

4. There are Two Components of Every Crime


o Objective: Actus Reus- The Physical and Voluntary act of the crime
A. However, theres a minimal mental element required in order to establish
the Actus Reus itself. The element of volition (voluntary action).
o Subjective: Mens Rea- The Mental & criminal intent/knowledge to commit act.
HYPOTHETICAL: Officer Blue and the killing of two innocent Vulcan teens.
Even in the Actus Reus is satisfied, the nature of this killing and the reasonableness of his
responses, is a jury question.
Action: Actually doing something

Omission: Failing to act/do something when you had a duty/obligation to do so.

B. Omissions (Negative Acts)


2. Distinguishing Acts From Omissions
Barber v. Superior Ct- Dr. did a surgery. Pt. got worse. Dr. pulled plug w/consent.
o Issue: Is a physician under a legal duty to continue futile life-sustaining support
absent objection from a spouse and does the withdrawal of such life-sustaining
support with the consent of a spouse support a charge of murder? No. (Rule ^)
o Holding: No, There is no criminal liability for failure to act unless there is a legal
to duty to do so. Here, D had no legal duty to continue to provide treatment to P
once it was proved to be ineffective or futile in the opinion of qualified
physicians. Ds omission to continue life-sustaining treatment to P under the
circumstances, though intentional and with knowledge that he would die wasnt
an unlawful failure to perform a legal duty.
There is no criminal liability for failure to act unless there is a legal duty to act.
o Duties arise in cases involving:
(1) Status Relationship
(2) Contractual obligation
(3) Omissions following an act and
(4) Statute.

Chapter Five- Mens Rea


B. General Issues in Preventing Culpability
1. Intent
o People v. Conley- D tried to hit one kid, but hit another & severely injured him.
Issue: May a court infer the intent to commit a crime based on
surrounding circumstances? Yes.
Rule: Criminal intent may be inferred from the circumstances surrounding
a crime.
Holding: Yes, The relevant statute requires that a person intentionally or
knowingly inflict permanent disability or disfigurement. After defining
Intent and Knowledge below, this statute requires that someone act with
the intent to inflict the requisite harm or be almost certain that the requisite
harm will occur.
Intent- When one intentionally acts to accomplish a certain outcome or conduct.
Although proving intent may be hard, the law presumes that one intends
the natural and probable consequences of their actions, and they Must
have the Mens Rea to intend the alleged crime.
o Intent to harm can thus be inferred from the circumstances surrounding a persons
actions, such as the words spoken or weapons used.
Knowledge- Being almost certain that something will occur as a result of an action.

Doctrine of Transferred Intent: Transferred intent is a doctrine that allows the


defendant to be held liable for an intentional tort he intended to commit against A but,
instead, accidentally committed against B.
3. Knowledge of Attendant Circumstances (Willful Blindness Test)
State v. Nations- Underage girl found dancing at a club for tips & owner didnt know.
o Issue: When a statute requires knowledge of a fact, as an element of a crime, is it
sufficient to prove the defendant was aware of the high probability of that facts
existence? NO.
o Rule: Where a statute requires knowledge of a fact as an element of a crime, it is
insufficient to prove the defendant was aware of the high probability of that facts
existence, unless the statute provides otherwise.
o Holding: The term knowledge, unless otherwise defined by statute, means
actual knowledge. When a criminal statute requires knowledge of a fact as an
element of a crime, the defendant must have had actual awareness of the fact to
satisfy that element.
o KNOWINGLY is defined as actual knowledge in our Criminal Code.
This definition Excludes those cases in which the fact would have been
known, had not the person willfully shut his eyes to avoid knowing.
The Doctrine of Willful Blindness: Defendant cannot escape criminal liability for
having knowledge of a fact by deliberately shielding him/herself from clear evidence of
critical facts that are strongly suggested by the circumstances.
o The purposeful ignorance of a fact that is highly likely to be true.
C. Strict Liability Offenses
Staples v. U.S.- D owned a rifle & after modifying, it qualified as a firearm. Ds unaware.
o Issue: May a person be convicted under the National Firearms Act, 28 U.S.C.
5861(d), if she did know that she possessed a weapon that fell within the statutes
registration requirement? No.
o Rule: Absent a clear statement from Congress that there is no mens rea
requirement, federal felony statutes should not be interpreted so as to eliminate
the element of Mens Rea.
Public policy disfavors criminal statutes with no mens rea
requirement
Strict Liability- The legal responsibility for damages, or injury, even if the person found
strictly liable was not at fault or negligent.
What is the Presumption Against Strict Liability?
o Offenses that do not contain a mens rea requirement have a generally disfavored
status.
HYPOTHESIS- SneakyMaxHypo:

Strict liability StatuteLAST BULLETPOINT ON LAST SLIDE FROM STAPLES V. UNITED STATES

HYPOTHESISMyHusbandisDead.(OldTestQuestion)
Under definition of intent, she is guilty of killing her husband and the other patrons.
Oh Roger Hypo: OLD TEST QUESTION
o Specific intent
o Not guilty of Larceny, because he said he was intending to BORROW & return
the clothes before his neighbor got home.
D. Mistake and Mens Rea
1. Mistake of Fact
o People v. Navarro- Stole wood beams with good faith that they were abandoned.
Issue: Is a mistake of fact a defense to a specific intent crime even if the
mistake was unreasonable? Yes.
Rule: An honest mistake of fact is a defense to a specific intent crime
regardless of whether the mistake was unreasonable.
o If, due to ones Honest Mistake of fact, a person is incapable of possessing a
specified mental intent, that person Cannot be guilty of that crime. This is the case
even if the defendants mistake of fact is unreasonable, as long as the defendant is
sincere in his mistake.
Specific intent crimes- A crime where the individual must not only intend to perform the
criminalized act, but also must intend the specific consequences of that act. (Require a
person to possess a particularized mental intent)
o D is not guilty of an offense if a mistake of fact negates the specific-intent portion
of the crime, even if mistake is unreasonable.
General intent crimes- A crime for which the prosecution must prove only that the
defendant intended to commit the act, not break the law. (Where a specific mental
intent is unnecessary.)
D is not guilty of the offense provided, the mistake of fact is reasonable.
o A Good faith belief that is reasonable is not a defense in General Intent crimes.

2. Mistake (or Ignorance) of Law

People v. Marrero- May one who violate a statute raise a good faith mistaken belief as to the
meaning of the law as a defense? NO. (Negated Issue = Rule)
Mistake/Ignorance of law
o CL Maxim- Ignorance of the law is no excuse.
What is the difference between a mistake of law and a mistake of fact?

o Mistake of Law- A misconception that occurs when a person with complete


knowledge of the facts reaches an erroneous conclusion as to their legal effect; an
incorrect opinion or inference, arising from a flawed evaluation of the facts.
o Mistake of Fact- An error that is not caused by the neglect of a legal duty on the
part of the person committing the error but rather consists of an unconscious
ignorance of a past or present material event or circumstance or a belief in the
present existence of a material event that does not exist or a belief in the past
existence of a material event that did not exist
In Criminal law, a Mistake of Fact can usually operate as a defense so
long as it is reasonable. With crimes that require specific intent, even an
unreasonable mistake of fact might work as a defense.
What Circumstances can a Defendant Claim Mistake of Law?
o Mistake founded on official statement of law in statute.
o Officially made/issued by public servant/agency/body legally charged with
administering/enforcing it.
Cheeks v. U.S- Part of an Anti-Tax advocacy group & charged w/ not filing a Fed. Tax Return.
Issue: Must a purportedly good-faith misunderstanding of the law be objectively
reasonable to negate the specific intent requirement of willfulness under criminal tax
laws? No.
o Rule: Whether a purportedly good-faith misunderstanding of the law will negate
the specific intent requirement of willfulness under criminal tax laws is a question
of fact for the jury; there is no legal requirement that the belief be objectively
reasonable.
How does the term willfully determine the outcome of the crime?
o Knowledge of the law is an element of the crime. Mens Rea. I intend to do
something but I did not willfully violate the law.
What mistake did the trial court make?
o Not giving the instruction on the willfulness knowledge/defense to the jury.

CHAPTER SIX- CAUSATION


A. Actual Cause (Cause in Fact)
Velazquez v. State- Two dragracers, brakes didnt work in time and P went over the cliff
o Issue: Is a defendants conduct the proximate cause of a prohibited result where
the result is beyond the scope of the defendants conduct or it would otherwise be
unjust to impose criminal liability? NO.
o Rule: Even where a defendants conduct is a cause-in-fact of a prohibited result, it
is not the proximate cause if the prohibited result is beyond the scope of the
defendants conduct, or it would be unjust to impose criminal liability.
For a crime, you need Actual cause and Proximate cause.

10

**You cant be the proximate (legal) cause without being the actual cause BUT we
can be the actual cause without being the proximate (legal) cause.*
Actual Cause (Cause in fact)- Actual cause refers to a cause or factor without which the
event could not have occurred. It is also termed as but for cause or cause in fact or
factual cause. The but-for test is often used to determine actual causation

Distinguish between: Application of the But For Test & Substantial Factor test
o But For Test- Defendants conduct is a cause-in-fact of a particular result if the
result would not have happened in the absence of D conduct.
o Substantial Factor Test- Defendants conduct is the cause in fact of a prohibited
result if the subject conduct was a substantial factor in bringing about the harmful
or deadly result.
The substantial factor test is important in injury cases. For example, if a
plaintiff works in a factory and develops cancer, he might allege that the
cancer resulted from asbestos poisoning. The defendant factory owner will
likely question whether the factory's asbestos was a substantial factor in
causing the cancer or whether other factors played a far more significant
role.
For good reason, many courts have declined to impose criminal liability
o Where the prohibited result of the defendants conduct is beyond the scope of any
fair assessment of the danger created by the defendants conduct, or
o Where it would otherwise be unjust, based on fairness and policy considerations,
to hold the defendant criminally responsible for prohibited result.

B. Proximate Cause (Legal Cause)


People v. Rideout- Car accident. All were safe. P went to turn on lights. Got hit by a car.
o Issue: Can a defendants conduct be the proximate cause of a victims injury
when there is another intervening cause?
o Rule: A defendants conduct is the proximate cause of a victims injury if it is a
direct and natural result of the defendants actions.
A person or event cannot be the proximate cause of harm unless she or it is an actual
cause, but a person/event can be actual cause w/out being proximate cause.
Proximate (Legal) Causation- an act from which an injury results as a natural, direct,
uninterrupted consequence and without that act, the injury would not have occurred.
o Proximate cause is the primary cause of an injury.
o Ds conduct must be the direct and natural result of Vs injury? To determine:
1. Was there an intervening cause that superseded Ds conduct, such that
is severed the casual link between Ds act & Vs harm? If no, Proximate.

11

An intervening cause severs the casual link when it is not reasonably


foreseeable under an objective standard. *Gross negligence and
intentional misconduct are not reasonably foreseeable.
Negligence is foreseeable at a hospital but GROSS neg. is not.
Actual (Factual) Causation- is but-for the action committed the outcome would not
have naturally occurred. (if you hadn't knocked over the candle there would have been no
fire)
o Factual causation determines exactly what happened and whether someone was
responsible for the event or whether it would have occurred anyway.
Actual Cause = Facts of the crime.
Proximate Cause= Legal aspect of the crime.
o Cant have a legal (proximate) cause without the facts (actual cause), BUT, you
Can have actual cause (facts of case) without any legal (proximate) liability.
Apparent Safety Doctrine- When a defendants active force has come to rest in a
position of apparent safety, the court will follow it no longer.
Responsive Intervening Cause- Harm results from actions taken in response to Ds
conduct (Ex: Surgery or Medical attention) Will establish proximate cause
Coincidental Intervening Cause- Wont establish proximate cause unless it was
foreseeable.
Superseding Intervening Cause- Does not need to be the only cause.

HYPOTHETICALMikeandAaronPunch/BankRobbery
o Who is the proximate cause of Aarons death? Bank Robber. (Answer on Twen)
HYPOTHETICALRalph
o He was not the but for factor and you should apply the Substantial Factor
Test.
HYPOTHETICALElevator
o Suri is the Factual cause but not the legal cause.
HYPOTHETICALApril
o Who was the actual cause of Aprils death? Maggie
o Who was the proximate cause of Aprils death? Maggie
o Responsive or Coincidental cause? Responsive

CHAPTER SEVEN- CRIMINAL HOMICIDE


A. Overview
4. The Protected Interest: Human Being
People v. Eulo
o Issue: In homicide proceedings, may courts determine when death occurs using
brain-based criteria rather than the traditional cardiorespiratory criteria? Yes.

12

o Rule: In homicide proceedings, courts may use brain-based criteria in addition to


traditional cardiorespiratory criteria as a means of determining when death occurs.
PracticeExamQuestions
Question1:
Must mention that this was a specific intent crime. (Issue)
Whether or not he had a good faith belief, not that his beliefs were unreasonable.
o Good Faith- if something makes no sense, but you truly believed you werent
committing a crime
o Unreasonableness- youre just an unreasonable person which leads to an
unreasonable mindset.
This case is all about detail. Noticing something that will completely switch up outcome.
o A six year old has no criminal responsibility in the United States. That fact must
be taken under consideration and touched on as a potential issue.

Question2:
He would be found guilty.
Mens Rea does not matter, even though he didnt mean to drink the alcohol.
Essentially, he had the Malition to move his body, and he did so by driving.
B. Intentional Killings
1. Degrees of Murder: The Deliberation-Premeditation Formula
o State v. Guthrie- Dishwasher snapped on instigator and killed him on impulse.
Issue: To constitute first-degree murder, must the defendant have had
some period of time between the development of the intent to kill and the
actual killing to indicate that the act was premeditated and deliberate and
not impulsive? Yes, R-there must be some opportunity for reflection on
the intention to kill after the intent is formed by the accused. (Failed to
inform jury of difference between 1st and 2nd degree murder)
Murder- The killing of a human being by another human being with malice aforethought
Malice Aforethought- A predetermination, or intent, at the time of a killing to willfully
take the life of a human being or an intent to willfully act in callous and wanton disregard
of the consequences to human life.
Malice includes these 3 elements:
o 1. Intent to kill
o 2. Intent to inflict grievous bodily injury
o 3. Extreme recklessness (deprave heart); & intent to commit felony where death
results.
First Degree Murder (Prior Consideration)- Intentional and unlawful killing of a human
being with malice, premeditation and deliberation. An elaborate plan or scheme is not

13

needed, only that there is evidence that the defendant considered and weighed his
decision to kill.
Second Degree Murder (Spur of the Moment)- Which may only require the intent to
kill. Any other spontaneous, but intentional, killing is second-degree murder.
Difference between 1st and 2nd degree murder?
1st Degree- Prior Consideration even smallest amt. of time is too much
2nd Degree- Spur of the Moment there was no though put into it.
Doctrine of Lesser included Offenses- D can be found guilty of an offense included
within the offense charged, as long as fact finder can reasonably conclude such from the
evidence introduced at trial (2nd degree murder rather than 1st degree murder)
Impulse v. Careful reflection
Impulse Careful Reflection
State v. Foresto Issue: Does a conviction for murder in the first degree require substantial
evidence of premeditation and deliberation?
o Rule: A conviction for murder in the first degree requires substantial evidence of
premeditation and deliberation, which may be proven through circumstantial
evidence.
There can be intent, even if your motives are honorable and justifiable.
Circumstantial Evidence- Evidence not based on personal observation or knowledge.
There are 6 Factors to Determine whether a Killing was Premeditated and Deliberate.
o (1) A lack of provocation from the victim;
o (2) The actions and words of the defendant before and after the killing;
o (3) Any threats on the part of the defendant before or during the killing;
o (4) Whether the victim and the defendant had a poor history;
o (5) Whether there was an additional lethal attack after the victim was already
helpless; and
o (6) Evidence of brutality
HYPOTHETICAL: The Missed Shot
o Murder is the killing of a human by a human, and Momo killed a dog.
o However, if an animal kills a human, the owner could be prosecuted for murder.
B. Intentional Killings
2. Manslaughter: Heat of Passion Killings
A. Common Law Principles
Understand the Distinction between Murder and Manslaughter
o Murder- The unlawful killing of a human being with malice aforethought.
o Manslaughter- The unjustifiable & intentional killing of a human being without
deliberation, premeditation, and malice.

14

o Voluntary Manslaughter- An intentional homicide, done in a sudden heat of


passion, caused by adequate provocation.
Girouard v. Stateo Issue: Is a verbal domestic argument ending with the death of one spouse
adequate provocation to mitigate murder to manslaughter? NO. (not adequate)
For provocation to be adequate it must be calculated to inflame the passion of a
reasonable man and tend to cause him to act for the moment from passion rather than
reason.
Adequate Provocation- Calculated to influence the passion rather than the reason. (i.e.
discovering a spouse in act of actual interference.)
What constitutes the Rule of Provocation?
o 1.) Adequate provocation
o 2.) Killing in the heat of passion
o 3.) Sudden heat of passion/no reasonable opportunity to cool down
o 4.) Casual connection between provocation/passion/act
Words constitute adequate provocation if they are accompanied by conduct intending a
present intention and ability to cause D bodily harm. But generally, do not amount to
provocation.

HYPOTHETICAL: Provocation- Court didnt allow adequate provocation. They relied on the
common law definition and rulings on words and said it didnt satisfy the elements.
D. Unintentional Killings: Unlawful Conduct
1. The Felony Murder Rule
The Doctrine in Generalo The Felony-Murder rule permits severe punishment for the most heinous offenses
in some cases that can appropriately be described as accidents. In its classic form,
the operation of the rule follows a compellingly simply. Almost mathematical
logic:
o A felony (during the commission of rape, robbery, burglary, mayhem, or lewd
acts with a minor) + an unlawful killing = Felony- 1st degree murder (strict
liability)
Death from unspecified felonies= 2nd degree murder
A classic formulation of this doctrine declares that one is guilty of murder if a death
results from conduct during the commission or attempted commission of any felony.
THECONCEPTUALBASISOFTHEFELONYMURDERRULE
Does the Felony Murder Rule Apply?
o Is it Inherently Dangerous?
Abstract or Fact of Case Test?
In the Abstract Test- elements without consideration of applicable facts;
Facts of the Case Test- Manner and circumstances in which it was actually committed.

People v. Fuller:
15

Issue: Is a death resulting from the commission of a dangerous felony considered to be


murder? YES.
Rule: Under the felony-murder rule, a death resulting from the commission of a
dangerous felony is murder.
o An accidental death can still be charged as a felony murder.
Holding: Yes. Under the felony-murder rule, if a killing results from the perpetration or
attempted perpetration of certain felonies enumerated by statute, the perpetrator is guilty
of first degree murder, even if the killing is negligent or accidental. In California, such
felonies include arson, rape, robbery, burglary, mayhem, and lewd acts with a minor.
(California Penal Code Sec. 189).
Can a co-felon be convicted for the unintentional act of killing by the other co-felon?
o Yes, they can.
Is there a requirement that the felony be dangerous to human life?
o No, there is not.
Is it necessary to prove malice?
o No, its not.
II. The Inherently Dangerous Felony: Limitation
People v. Howard
o Issue: Is a felony inherently dangerous even if it can be committed without
creating a substantial risk of death to another? No, because of new legislation.
o Holding: No. In California, a killing resulting from the commission of an
inherently dangerous felony is at minimum a second- degree murder. Only
felonies that create a substantial risk of death to another can serve as the basis of a
felony-murder charge. The elements of the felony must be inherently dangerous in
the abstract.
Murder is the unlawful killing of a human being with malice aforethought.
o **But under the second-degree felony-murder rule, the prosecution can obtain a
conviction without showing malice if the killing occurred during the commission
of an inherently dangerous felony.**
Understand the Inherently Dangerous Felony Limitation v. Felony Murder Rule.
When is a felony inherently dangerous?
Prosecution can obtain a conviction for 2nd degree felony murder if killing occurred
during commission of an Inherently dangerous felony.
o Court applies two tests:
In the Abstract Test- elements without consideration of applicable facts
Conviction of a felony murder is very unlikely under this test.
Facts of the Case Test- Manner and circumstances in which it was
actually committed.
o Court reasons, that if there is any way the felony can be committed without
creating a substantial risk of death to someone, it is not Inherently Dangerous.
BABY SARAH HYPOTHETICAL- (ABSTRACT TEST APPLIES)

16

Abstract Test- Not necessarily guilty because anyone under the age of 18 is included.
Without the facts, we could assume the child was a 17 year old, home alone after school, and
snuck one of Dads beers while making inappropriate gestures at the neighbors. This
wouldnt lead us to believe its a felony.
Facts Test- Once we apply the facts, we see that it was a helpless child in this case and that it
should in fact be a felony.
A FAILED HEART HYPOTHETICAL- (FACTS TEST APPLIES)
(Felony murder rule will apply to attempted felonies as well)
Margaret is guilty because the doctrine declares that one is guilty of murder if a death
results from conduct during the commission or attempted commission of any felony.
Robbery is considered a felony, and she was attempting to commit the robbery when
the plaintiff had a heart attack and died. Margaret would be convicted.

III. The Independent Felony (Merger Limitation)


People v. Smith: Beat her daughter until she went into Card. Arrest and died. Smith Charged
with 2nd deg. Murder based on a theory of felony-murder.
Issue: Can a felony that is assaultive in nature serve as the basis of a felony-murder
charge? No, because the assaultive felony is said to merge with the murder. Smiths
abuse of her daughter merges with the resulting homicide so that Smith cannot be
charged with felony-murder. The jury must instead determine whether Smith possessed
the requisite malice for a conviction of murder. (Child abuse is assaultive in nature and
has no other independent felonious purpose)
1. The Jury was instructed that they could find the defendant guilty of second-degree
felony murder if it determined that the homicide occurred during the commission of
the underlying felony of assault with a deadly weapon.
Rule: A felony that is assaultive in nature cannot serve as the basis of a felony-murder
charge unless the felony was committed with an independent felonious purpose.
Merger Doctrine- Felony Murder only applies if the predicate felony is independent of or
collateral to, the homicide. If the felony is not the independent cause, then the felony is said to
merge with the homicide and cannot serve as the basis for a felony-murder conviction.
Defendant may only be charged with the higher offense.
1. IFL typically applicable to Assaultive felonies (threat of immediate bodily injury)
Fully understand the Independent Felony Limitation*.
PETE HYPOTHETICAL
Cant be charged with the Felony Murder Rule because the heat of the passion merges
with the homicide or manslaughter charge. There was no malice for the prosecutor to
prove in this case which means that since no murder resulted from the commission of a
felony, therefore making it impossible to convict under this rule.
17

VERY IMPORTANT CASE/INFORMATION!!


IV. Killings in the Perpetration or in Furtherance of a Felony
State v. Sophophone
Issue: Is the felony-murder rule applicable when a non-felon performs the killing
lawfully? NO. (Its not applicable)
Holding: No. The felony-murder rule is not applicable where a non-felon is responsible
for the resulting death. [B: Agency Approach]. Under this approach, the act of killing is
imputed to the defendant when committed by an accomplice.
CLASS NOTES
Fully understand the Res Gestae Doctrine (begins when the actor has reached a point
where prosecution for attempt is appropriate), AS WELL AS:
The Agency Approach
o Felony-Doctrine doesnt apply if person who caused death is a non-felon (The cofelon is not an accomplice of the killer). (Majority Rule)
The Proximate Causation Approach
o Felon would be responsible for killing by non-felon if felon set in motion the acts,
which resulted in victims death.
The Applicable Provisions of K.S.A. 21-3436 state:
o Murder in the first degree is the killing of a human being committed
(b) in the commission of, attempt to commit or flight from an inherently
dangerous felony as defined in K.S.A. 21-3436 and amendments thereto.
Aggravated burglary is one of the inherently dangerous felonies as
enumerated by K.S.A. 21-3436(10).
Defendant doesnt argue that aggravated burglary is an inherently dangerous felony. His
principal argument is that he was in custody at the time his co-felon was killed.
o He argues that this was a break in circumstance sufficient to relieve him from
further criminal liability.

This Intervening Cause argument has no merit under the facts of this case.
o In many similar cases, it has been held that time, distance, and the casual
relationship between an underlying felony and a killing are factors to be
considered in determining whether the killing occurs in the commission of the
underlying felony and the defendant is therefore subject to the felony-murder rule

Two Approaches to decide whether to apply the Felony-Murder Rule:


The Agency Approach
o The majority opinion is that the felony-murder doctrine doesnt apply if the
person who directly causes the death is a non-felon.
The Proximate Causation Approach

18

o An alternative theory holds that a felon may be responsible under the felonymurder rule for a killing committed by a non-felon if the felon set in motion the
acts, which resulted in the victims death.
Pursuant to this rule, the issue becomes one of proximate cause:
If an act by one felon is the proximate cause of the homicidal
conduct by the non-felon or police officer, murder liability is
permitted.

HYPOTHETICAL- FRANK AND JAMES


o Frank wouldnt be found guilty under the Felony-murder rule under the Agency
Approach. He was a non-felon and engaged in an attempt at self-defense.

HYPOTHETICAL- SIMI
o Simi was unaware that the robbery was even happening, so it had no causal
connection with his death. There must be a casual relationship between the
felony and the killing.
o One argument that establishes causation- If you can prove that his death had
nothing to do with the robbery, but had it not been for the robbery, people would
have normally been around Simi when he had the heart attack, and possibly
couldve prevented it. there is a causal connection and the outcome could change.

HYPOTHETICAL- OH NO! POLICE OFFICER FELL INTO AN AIRSHAFT


o Prosecution/Officer would prevail on applying the felony- murder rule because
had the officer not been in a lawful pursuit of the defendant, he would not have
been chasing him on the roof or fallen into an airshaft and died. Momo is not the
proximate or actual cause of the officers death, but there was still a causal
connection. Momos negligence of his death. Momo would lose. (There was a
death but no killing.)
E. Capital Murder
2. The Quest for Reliable Procedures
B. Victim Impact Evidence
Payne v. Tennessee
o Issue: May juries hear victim impact evid. at a capital sentencing hearing? YES.
o Rule: A capital sentencing jury may hear victim impact evidence if it is relevant
to the jurys decision as to whether the death penalty should be imposed
o
3. Substantive Limitations on the Death Penalty
Tison v. Arizona
o Issue: Is it constitutionally permissible to sentence a defendant convicted of
felony-murder to death where the defendant neither intended to kill the victim nor
actually inflicted the fatal injury? YES.

19

o Rule: It is constitutionally permissible to sentence a defendant convicted of


felony-murder to death even if the defendant neither intended to kill the victim
nor actually inflicted the fatal injury if the defendant was a major participant in
the felony and possessed a reckless indifference to human life.
CHAPTER EIGHT- RAPE
B. Forcible Rape
1. In General
State v. Alston
o Issue: In order to prove rape, is it sufficient to show that sexual intercourse was
obtained without the consent of the victim? NO.
o Rule: In order to prove rape, there must be sufficient evidence that sexual
intercourse was obtained against the victims will and by force.
Holding: No. Second-degree rape requires that the defendant obtain sexual intercourse
both by force and without the victims consent. Force is demonstrated by actual force or
by threats that are intended to overcome the will of the victim to resist. Consent can be a
complete defense to rape, unless it is obtained by force or threat of force. Where a
defendant is engaged in a consensual sexual relationship with the victim, the victim can
freely revoke her consent at any time. Such evidence demonstrates that the sex was
against her will.
CLASS NOTES
Understand Forcible Rape
Rape is the carnal knowledge of a woman forcibly and against her will CL
Force or threat of physical force is required
Penetration to any degree is sufficient
Physical resistance is not required
Consent by the victim is a complete defense
o Consent that is induced by threat of violence is void.
o General fear of a defendant is not sufficient to void consent
C. Rape by Fraud or Non-Physical Threats
Boro v. Superior Court
o Issue: Does rape obtained by fraud in the inducement nullify consent?
o Rule: Rape perpetrated by fraud in the factum nullifies consent, whereas rape
perpetrated by fraud in the inducement does not.
o Rape perpetrated by fraud in the factum nullifies consent, whereas rape
perpetrated by fraud in the inducement does not. Fraud in the factum is a
misrepresentation of a fact that goes to the very nature of that fact.
CLASS NOTES

20

Understand the distinction between Fraud in the Factum and Fraud in the Inducement
in rape cases.
Factum- when the deception (fraud) causes a misunderstanding as to the fact (sex act)
itself, there is no legally recognizable consent because what happened is not that for
which consent was given.
o Example: patient that consents to penetration by a medical instrument, not to
sexual intercourse.
Inducement: where consent to sexual intercourse is induced by deception (fraud),
consent is valid, absent statutory language to the contrary.
o Example: Victim consents to sex with a single, wealthy man, but in actuality he is
married and homeless, but deceives victim into believing otherwise.

CHAPTER NINE- GENERAL DEFENSES TO CRIMES


C. Principles of Justification
2. Self- Defense
United States v. Peterson
o Issue: (1) May the initial aggressor in a fatal conflict invoke the doctrine of selfdefense to justify killing his adversary? NO.
o Rule: (1) Under the law of the District of Columbia, the initial aggressor in a fatal
conflict may not invoke the doctrine of self-defense to justify killing his
adversary, unless he withdrew from the conflict in good faith and communicated
his withdrawal by words or acts.
The doctrine is rooted in necessity. Deadly force is only justified if there is
no alternative. A person must honestly and reasonably believe that there is
an imminent threat of death or serious injury. Only then may he use deadly
force to save himself. Nevertheless, a person who provokes a fatal conflict
does not have a right to kill in self-defense.
o Issue: (2) Is the initial aggressor in a fatal conflict under a duty to retreat before
using deadly force in self-defense? YES.
o Rule: (2) Under the law of the District of Columbia, the initial aggressor in a fatal
conflict is under a duty to retreat, if he may do so safely, before using deadly force
in self-defense.
CLASS NOTES
Fully Understand:
1.) When one has a right to self defense (cant support it w/ self-generated Necess. 2 kill)
2.) Who is the Aggressor? (must commun. intent 2 withdraw & attempt in good faith)
3.) Duty to Retreat (deadly force not authorized when one can safely retreat w/o risk)
VS. Castle Doctrine (one attacked in home is not required to retreat)
4.) NPR= Necessity (imminent), Proportionality (non-deadly= non-deadly force),
Reasonable Belief.

21

B. Reasonable Belief Requirement


People v. Goetz
o Issue: Is a person justified in using deadly force in self-defense if he subjectively
believed such force was necessary to prevent an attack or a robbery? NO.
o Rule: In New York, a person is justified in using deadly force in self-defense or
defense of another only if she objectively and reasonably believes an attacker is
either (1) using or about to use deadly force or (2) committing or attempting to
commit a kidnapping, forcible rape, forcible sodomy, or robbery.
CLASS NOTES
What is the Reasonable Belief requirement in self-defense case?
Subjective and Objective components must be met; actual outcome is irrelevant
o Subjective-The defendants had a good faith subjective belief that something under
this category was about to happen.
o Objective- a person under the same circumstances as the person who used the deadly
force wouldve reacted the same way. It would have been reasonable for anyone
under those circumstances to react the same way this person did.
You can satisfy subjective/objective components w/o the other.
A person may not use deadly force upon another person unless an attacker is either (1)
using or about to use deadly force or (2) committing or attempting to commit a
kidnapping, forcible rape, forcible sodomy, or robbery.
HYPOTHESIS- MAGGIE FROM ITALY:
Subjective- This component is met because she was in good faith Fearful for her life
Objective- List all of the objective things that would lead her to believe shes in danger.
o They all looked directly at Maggie.
o No aggressive conduct= she didnt meet the requirements for a self-defense claim
based on the reasonable persons approach.
HYPOTHESIS- DONT SLAP ME:
o She would not prevail in a self-defense claim. You cant use deadly force for nondeadly force conduct.

General Defenses to Crimes


C. Principles of Justification
3. Defense of Others
People v. Kurr
o Issue: Is a defendant who kills in the defense of an unborn and unviable fetus
entitled to a jury instruction on the defense of others? YES.
A NON-VIABLE FETUS IS ALWAYS WORTHY OF PROTECTION

22

o Rule: A defendant who kills in the defense of an unborn and unviable fetus is
entitled to a jury instruction on the defense of others.
o Holding: Just as a homicide is justifiable if it is committed in self-defense, a
homicide is also justifiable if it is committed in the defense of others. In this
jurisdiction, the defense applies to a fetus, regardless of whether it is viable.
Michigans legislature has passed the fetal protection act, which demonstrates its
intent to protect fetuses, both viable and unviable.
However, this defense is only applicable where the harm against the fetus
is unlawful. Thus, a defendant may not invoke this defense to justify a
homicide perpetrated to prevent a legal abortion.
o Legality of the act- this defense can only be used against unlawful assaulted or
negligent conduct. Never lawful conduct.

CLASS NOTES
o The intervener typically has the same right as the third party.
o Deadly force can be responded to with deadly or non-deadly force
o Non-deadly force can only be responded to with non-deadly force
o Understand Principles of Justification: Defense of Others
o Generally, a person is justified in using force to protect a 3rd party from unlawful
force by an aggressor. The interveners right to use force parallels the 3rd partys
right of self-defense.
o Use of the Defense of Others justification makes no distinction between
strangers and relatives.
o The defense may be evoked in the protection of an unborn child, including a
nonviable fetus or embryo, from assaultive or negligent conduct.
o Know the difference between alter ego & reasonable app. Rule
Defense of Another:
o The right of a person to protect a third party with reasonable force against another
person who is threatening to inflict force upon the third party.
"Alter Ego" Rule:
o The primarily obsolete rule that a person coming to the aid of a third party has no
more right to defend the third party than the third party would, himself, have had
to defend himself.
Reasonable Appearance Rule
o One reasonably believed that they were in danger and their reaction was proportional.
Also, he only used the force necessary to stop the attack.
HYPOTHETICAL- ZINA
After doing what he thought was saving Zina from a rapist, the man attacking her turned
out to be an undercover FBI agent and Rob was charged with assaulting the officer. Rob

23

was from Michigan, where they adopted the Alter Ego Rule & Illinois has adopted the
Reasonable Appearance Rule. Which one should Rob choose to be prosecuted under?
He should choose the Reasonable Appearance Rule because Intervener only has rights
that are afforded to the third party. Since she didnt lawfully have the right to resist the
lawful arrest, neither did he
o Under the Reasonable Appearance Rule, he reasonably believed that she was in
danger and his reaction was proportional. Also, he only used the force necessary
to stop the attack.

General Defenses to Crimes (Cont.)


D. Principles of Excuse
2. Duress
United States v. Contento- Pachon
o Issue: Is a defendant excused from criminal culpability if he commits his crime
under a threat of death or serious bodily injury? YES.
o Rule: A defendant is excused from criminal culpability if he commits his crime
under a threat of death or serious bodily injury.
A defendant is excused from criminal culpability if he commits the crime under duress.
There are three elements to the defense of duress.
o First- the threat of death or serious bodily injury must be immediate.
o Second- the defendant must act on a well-grounded fear that the threat will be
realized.
o Third- there must be no reasonable opportunity for the defendant to escape. Here,
the district court held that the threat was not immediate because the threat would
only be acted upon after Contento-Pachons future failure to cooperate.
CLASS NOTES
Fully understand Duress and Necessity
Duress: Immediate threat of death or serious bodily harm, genuine fear, no escapability
o MAN MADE THREAT TO YOU OR YOUR LOVED ONE
Necessity: A person is faced with a choice of two evils and must decide whether to
commit a crime or an alternative act that constitutes a greater evil.
Two Evils
o First- BREAKING CRIMINAL LAW. SOCIETY BARS CONDUCT.
o Second- LOSS OF LIFE. SOCIETY BEARS BURDEN OF DEATH.

3. Intoxication

United States v. Veach


o Issue: Does intoxication negate the mens rea for a crime and thereby serve as a
defense to the crime? YES.
o Rule: Intoxication negates the mens rea of a specific intent crime and thereby
serves as a defense, but it does not negate the mens rea of a general intent crime.

24

o However, intoxication only negates the mens rea of crimes that require specific
intent. In order to determine whether intoxication is a defense to the crimes Veach
was convicted of, this court must determine whether they are specific intent or
general intent crimes. Accordingly, 115 is a specific intent crime.

CLASS NOTES
Fully understand when intoxication as a defense to a crime.
o When it negates the Mens Rea of a Specific Intent crime. (Never General Intent)
Distinction between Voluntary and Involuntary Intoxication
o Voluntary- Only a defense for Specific Intent Crimes.
o Involuntary- Can be a defense to both Specific and General Intent Crimes.
Effect on Specific Intent Crimes? Negates Mens Rea
Effect on General Intent Crimes? Generally, none.
Distinction between Coerced, Pathological, Intoxication by Innocent Mistake, and
Unexpected Intoxication.
o Coerced- Intoxication involuntary induced by reason of duress or coercion.
o Pathological- Substance triggers an unusual response due to a medical condition.
o Innocent Mistake- Unaware of substances content or tricked into taking it.
o Unexpected- Unexpected intoxication from taking a medically prescribed drug.
Involuntary Intoxication
Specific Intent Crimes- If a charged crime is a specific intent crime, meaning that the
criminal defendant must have had the specific intent to commit the crime in question,
involuntary intoxication can be a defense to criminal charges if it prevents the defendant
from forming the intent that is required.
General Intent Crimes- Involuntary intoxication can also be a defense to a general
intent crime if the defendant can establish that the involuntary intoxication acted similarly
to an insanity defense and prevented the defendant from understanding the nature of his
or her actions or differentiating between right and wrong.
Voluntary Intoxication
Unlike involuntary intoxication, voluntary intoxication is NEVER a defense to a general
intent crime. However, voluntary intoxication may be used as a defense to specific intent
crimes if, as with involuntary intoxication, it prevents the defendant from forming the
criminal intent necessary to commit the crime.
o In some cases, the defense of voluntary intoxication does not completely absolve
the defendant of liability but instead reduces the overall culpability for the crime.
MIDTERM QUESTION

25

Sarah is pregnant. Tom walks up to her with a gun and tells her once she has that mans baby in 3
months he will kill her. Tom looks away and Sarah stabs him in the neck killing him. Should she
win on self-defense grounds?

4. Insanity
C. Struggling for a Definition: The Tests of Insanity
State v. Johnson
o Issue: Whether the test for insanity should consider solely cognitive defects.
o Rule: The test for insanity should encompass both cognitive and volitional
defects.
MNaghten Rule- A rule that says a defendant is not criminally liable if, by reason of
mental disease or defect, he does not know the nature and quality of his actions or, if he
did know it, he did not know his actions were wrong.
CLASS NOTES
Fully understand the Insanity defenses (each requires mental disease or defect at the
time of crime according to legal rather than medical policy)
NEVER WILL TEST IRRESISTABLE IMPULSE/CONTROL TEST
The MNaghten Rule
o Clearly proved at time of committing the act, D was laboring under defect of
reason, from disease of the mind.
As not to know nature and quality of the act, or
If he did know it, did know that what he was doing wrong (legally)
(ALL cognitive test)
o Wont test on whether its a legal or moral wrong. Play it safe and go with both.
o MNaghten doesnt deal with Volitional deficiencies. ONLY Cognitive.
Difference between Cognitive and Volitional Deficiencies.
o Cognitive- Defect in my brain. Unable to decipher right activity from wrong.
o Volitional- I know what Im doing, and that its wrong, but I cant help but do it.
Irresistible Impulse/Control Test
o 1. Acted with irresistible repulse
o 2. Lost power to choose between right and wrong
o 3. Defendants will and actions are beyond their control
Product Test
o Accused not criminally responsible if his unlawful act was product of mental
disease/defect
Model Penal Code Test- Section 4.01 (acknowledges volitional and cognitive defects)
o When, as a result of mental disease or defect, defendant lacked substantial
capacity to appreciate criminality of conduct. OR,
o When, as a result of mental disease or defect, defendant lacked substantial
capacity to conform his conduct to requirements of law.

26

HYPOTHETICAL- I Cant Breathe


He does not satisfy the MNaghten rule because he doesnt acknowledge the moral factor,
but we can infer that we know he was merely playing.
As to the nature and quality of the act, he cant claim that he didnt know those facts
because he was squeezing a human beings neck. N&Q doesnt mean that he doesnt know
the level of pain or damage hes causing, but it goes to whether or not he was aware of
what he was actually doing. The fact that he answered yes to knowing he was choking a
human being is enough to prove that he knew the nature and quality of what he was
doing.
Nature- What is actually being done.
Quality- The impact of whats been done.

6. Infancy
In re Devon T
Issue: Is a child criminally culpable for his actions if he is incapable of differentiating
between right and wrong? NO.
Infancy Defense- Defense that removes liability for a juveniles crime if the juvenile
defendant is unable to differentiate between right and wrong.
The primary consideration in an infancy case is whether the juvenile defendant can
distinguish right from wrong.
o If the defendant can make the distinction, he is deemed criminally culpable for his
actions.
According to common law, children under seven years of age are incapable of
differentiating between right and wrong.
o Children fourteen or older are deemed capable.
Children between seven and fourteen are presumed incapable, but this
presumption is rebuttable.
o However, the closer the child is to the age of fourteen, the more likely he can
appreciate the difference between right and wrong, and the lighter the
prosecutions burden in overcoming the presumption.

CLASS NOTES
Fully understand the Infancy Doctrine- WILL BE TESTED ON THIS!
o For children (1) under the age of 7, the are presumed without criminal
capacity. Zero amount. 14 years and older are treated as fully
responsible; and (3) between 7 and 14rebuttable presumption of
criminal capacity.
The defense is available in Juvenile proceedings
Once defense is raised, the prosecutor has the burden of rebutting the
presumption by producing evidence that permits a reasonable inference that D
knew at the time of the act that the difference between right and wrong
(Cognitive Element ONLY/ no volitional).

27

Infancy Defense Percentage Rule- On day before 7th birthday, child has 0%
cognitive capacity. On day of 14th birthday, child possesses 100% capacity. On
the time scale between the 7th and day before 14th birthday the percentage of
children with capacity, steadily increases, so at midpoint, age 10 years, 6
months, 50% of children will have criminal capacity.
In re Devon Contd.
What are the potential flaws in the courts finding that:
Devon reaching the 6th grade while still 11 years old tends to support his
cognitive capacity.
That Devons silence and Stonewalling demeanor was an indication of his
allegiance to the Underworlds Code of Silence and that he is fully conscious
of the ongoing war between lawful authority and those that flaunt it.
Children who are unaware that what they are doing is wrong have no need to
hide out or conceal their activities.
E. New Defenses
2. Addiction/Alcoholism Constitutional Defense

Powell v. Texas- Powell was arrested for public intoxication. Powell argued that his
conduct was unavoidably caused by his disease of chronic alcoholism. He further argued
that punishing him for conduct that was symptomatic of his disease would constitute
cruel and unusual punishment, in violation of the Eighth and Fourteenth
Amendments. In support of his theory, a certified psychiatrist testify as to his condition.
Wade stated that although there is no clear definition of chronic alcoholism, one who
suffers from that condition drinks involuntarily. He further stated that Powell was a
chronic alcoholic who could not control his behavior because he has a strong compulsion
to begin drinking, and that once he begins drinking, he has an uncontrollable compulsion
to drink excessively. The trial court held that chronic alcoholism is NOT a defense to the
charge of public drunkenness. It did, however, allow Powell to submit three findings of
fact: (1) that chronic alcoholism is a disease that overpowers ones will to resist the
continuous and excessive consumption of alcohol; (2) that a chronic alcoholic who goes
out in public does so due to his disease, not out of free will; and (3) that Powell was a
chronic alcoholic. Powell was convicted of the charged crime.
o Issue: Whether punishment of conduct that is symptomatic of chronic alcoholism
is in violation of the Eighth and Fourteenth Amendments proscription against
cruel and unusual punishment. NO. (R- No, it is NOT in violation.)
o Powell is not punished for being a chronic alcoholic, but for breaking the law by
going out in public while intoxicated.
o Punishing conduct stemming from chronic alcoholism in NOT cruel and unusual
punishment because medical research on chronic alcoholism is too vague and
unsettled.
o If the Court rules that his chronic alcoholism relieves him of guilt, it creates a new
defense to criminal culpability.
28

o The punishment of conduct that is symptomatic of chronic alcoholism is not in


violation of the Eighth and Fourteenth Amendments proscription against cruel
and unusual punishment.

CHAPTER TEN- INCHOATE OFFENSES


B. Attempt
3. Mens Rea
People v. Gentry- Gentry lived with his girlfriend, Ruby Hill. On December 13, 1983,
Gentry and Hill had an argument, during which Gentry spilled gasoline on Hill. She later
went into the kitchen and was near the stove when the gasoline ignited. Gentry was able
to put the fire out, but Hill sustained serious burns. Gentry was tried for attempted
murder. The trial court instructed the jury on the definition of attempt murder, as well as
the four different mental states that were sufficient to prove murder. Gentry appealed,
arguing that the courts instruction as to the four different mental states allowed the jury
to convict him for attempt murder without showing that he had the specific intent to kill.
o Issue: Does conviction of attempted murder require a specific intent to kill? Yes.
o According to People v. Kraft, a finding of a specific intent to kill is necessary to
sustain a conviction for attempt murder.
o Mere knowledge that death or serious bodily harm may occur is insufficient.
Thus, the trial courts instruction listing the culpable mental states of
murder, which included both intent and knowledge, erroneously allowed
the jury to convict Gentry of attempt murder without finding a specific
intent to kill. Gentrys conviction is reversed and a new trial is ordered.
Inchoate Crime- A crime that is anticipatory or preparatory in nature and for which an
individual can be held criminally liable without the actual commission of the crime being
anticipated or prepared for.
4. Actus Reus
United States v. Mandujanoo Mere Preparation- An act that constitutes a step towards the completion of a
crime but does not constitute an overt act or a substantial step.
b. Distinguishing Preparation from Perpetration: The Tests at Work
People v. Rizzo- Rizzo along with three others intended to rob a man of a pay roll valued
around $1,200. Rizzo was to point out the man to the others who were to commit the
actual robbery. The men, two of whom had guns, drove around town in a car looking for
the man, but were never able to find him. During their search, nearby police became
suspicious and followed the vehicle. Shortly after Rizzo jumped out of the car and ran
into a building, all four were arrested for attempted robbery despite never finding the man
they intended to rob. Rizzo and the others were convicted of attempted first-degree
robbery and they appealed.
o Issue: Is the crime of attempt committed when an act as tending to the
commission of a crime is so near to its accomplishment that in all reasonable
probability the crime itself would have been committed, but for timely
interference? YES.

29

o Attempt is defined by New York statute as [a]n act, done with intent to commit a
crime, and tending but failing to effect its commission.
o The word tending is very indefinite. Tending means to exert activity in a
particular direction.
o Any act in preparation to commit a crime can be said to be tending towards its
accomplishment.
But it is only those acts which advance very near to the accomplishment of
the intended crime which support an attempt conviction.
Here, Rizzo and the others were looking for the pay roll individual to rob him of the
money, but they were never able to even locate him.
Rizzo and the others had the intent to commit the crime, but never had the opportunity.
Those acts are too remote to support the convictions for attempted first-degree robbery.
The judgments of conviction are reversed.

**ADD NOTES FROM SLIDES FOR GENTRY-RIZZO**

HYPO- Teachers Temptation


She should be charged.
Legal Impossibility occurs when Ds actions, even if fully carried out exactly as she
intends, would NOT constitute a crime.
Factual Impossibility occurs when the actions intended by D are criminals, but a
circumstance or fact unknown to the D prevents him from bringing about the intended
result. NO DEFENSE FOR ATTEMPT CHARGE.
ONE MORE SLIDE ON TWEN

State v. Reeves- Reeves (D) and Molly Coffman were twelve-year-old girls who attended the
same middle school. On January 5, 1993, Reeves and Coffman agreed over the phone to kill their
homeroom teacher, Janice Geiger, with rat poison. The next morning, Coffman took a packet of
rat poison to school. During the bus ride to school, Coffman told another student of the plan. The
student told school officials once she arrived at school. Geiger noticed when she arrived in her
classroom that Reeves and Coffman were leaning over her desk. They left a purse on Geigers
desk next to her coffee cup. Authorities found rat poison in the purse. Both Reeves and Coffman
were found guilty of attempted second-degree murder.
Issue- Whether a defendant is guilty of attempt only if the defendant was on the brink of
completing the offense. NO.
Rule- The crime of attempt does not require that the defendant is on the brink of
completing the offense.
Overt Act- An action that constitutes a manifestation of the desire to commit a crime
rather than just mere preparation for such an action.
Dupuy Rule- Defendant would have to be on the brink of completing the crime in order

30

to be found guilty of attempt. (Failed attempt) ABANDONED RULE.


Model Penal Code- if one purposely does or omits to do anything which, under the
circumstances as he believes them to be, a substantial step in a course of conduct
planned to culminate in his commission of the crime.
o If I believe that I have poison and if the facts are as I perceive them to be, I am
taking a substantial step towards harming/poisoning this person.
Holding- This court instead holds that if a defendant possesses materials for the
commission of a crime while at or near the scene of the crime, and the defendant has no
lawful purpose to possess the materials under the circumstances, a jury may rely on such
evidence to find that the defendant has taken a substantial step, if the evidence strongly
corroborates the actors criminal intent. The conviction is affirmed.

CLASS NOTES
Understand why the court abandoned the Dupuy rule.
When an actor possess materials to be used in the commission of a crime, at or near the
scene of the crime, and.
Where the possession of those materials can serve no lawful purpose of the actor under
the circumstances.
The jury is entitled, but not required to find that the actor has taken a Substantial Step
toward the commission of the crime if such action is strongly corroborative of the actors
overall criminal purpose.
People v. Thousand- On December 8, 1998, Deputy William Liczbinski made contact with
Chris Thousand (D) in an online chat room. Deputy posed as a 14 year-old girl named Bekka in
an effort to determine whether Thousand, a twenty-three-year-old male, was engaging in criminal
activity online. Over the next week, Bekka and Thousands conversations became sexually
explicit. Thousand sent her a photograph of male genitalia and asked if she wanted to meet in
order to engage in sexual activity. The two arranged to meet at a nearby restaurant. Once
Thousand arrived, Liczbinski apprehended him and charged him with attempted distribution of
obscene material to a minor. Thousand filed a motion to quash the charge of attempt, since it was
impossible to convict him of the underlying offense. The circuit court granted the motion and
dismissed the case. The Court of Appeals affirmed.
Rule- A defendant can be convicted of attempt even if it is impossible to complete the
underlying offense.
At common law, there are two types of impossibility: factual and legal.
o Factual impossibility- occurs when factual circumstances preclude the
completion of a crime.
Ex.- when a defendant tries to kill someone with an unloaded gun.
Factual impossibility is never a defense to an attempt crime.
o Legal impossibility- on the other hand, may be a defense to an attempt crime if it
is a pure legal impossibility. This occurs when a defendant believes his conduct is
criminal but in actuality, it is not.
Ex.- a defendant might engage in sexual intercourse with a fifteen year old
in a jurisdiction where the age of consent is fifteen.
Even if the defendant mistakenly believes the age of consent is
31

sixteen, he is not guilty of statutory rape.


o A Hybrid Legal Impossibility- occurs when a defendant is factually mistaken as
to a legal status, the existence of which is a necessary element of the crime.
Ex.- a defendant who receives unstolen property believing it is stolen may
argue that he cannot be charged with the crime of receiving stolen
property.
In that situation, he is factually mistaken as to the legal status of
the property.
Holding- It is true that Thousand cannot be convicted of the underlying crime, since he
could not be guilty of distribution of obscene material to a minor who does not exist. But
here, he is not charged with the underlying crime. He is charged with an attempt of that
crime, and it is irrelevant whether or not he could have completed the underlying crime.
All that is relevant is whether he had the requisite intent and took the requisite act in
furtherance of that intent. Accordingly, the circuit court should not have dismissed the
charge against Thousand.

CLASS NOTES
CONERNED WITH SUBJECTIVE BELIEF (what one believes) OF WHAT THE CASE
IS AND NOT WHAT IS ACTUALLY IS.
Factual Impossibility- NOT A VIABLE DEFENSE
o Exists when defendants intended end constitutes a crime but fails to consummate
it because f a factual circumstance unknown to her or beyond her control.
Legal Impossibility- DEFENSE
o Pure Legal impossibility
Bars an attempt conviction
o Hybrid legal impossibility
Defendants goal was illegal, but commission was impossible due to a
factual mistake made by her regarding the legal status of some factor
relevant to her conduct.
ONLY THING WE WILL BE TESTED ON IS
Distinction be tween factual impossibility and pure legal impossibility

Commonwealth v. McCloskey- Shortly after midnight, an alarm went off at Luzerne County
Prison, alerting guards that a prisoner was attempting to escape from the recreation area. The
alarm was only audible in the guards office. The guards immediately checked the inmates, but
no prisoners were found missing. Investigation revealed that someone had cut a piece of barbed
wire in the area where the alarm went off. The guards also found a bag filled with civilian
clothing. The guards later determined the bag belonged to McCloskey (D). McCloskey
approached one of the guards and confessed that he had planned on breaking out of prison that
night, but decided against it when he thought about the consequences. McCloskey was convicted
of attempted prison breach, and he appealed to the Superior Court of Pennsylvania.
Rule- A defendant is not criminally liable for attempt if his plans never go beyond mere
contemplation.
Holding-If the defendant crosses the line into intent and later abandons the plan, he may
be able to avoid liability for the crime, but not the attempt. See Eagan, supra. Here,
32

McCloskey intended to escape from the prison and made preparations accordingly.
Nevertheless, the evidence shows that McCloskey only got to the prison wall, which is
still inside the prison, before deciding to abandon his plans. McCloskey never reached the
point of actually carrying out his plan and thus never went beyond merely contemplating
the escape. Because McCloskey never set his plans in motion, he cannot be found
guilty of attempt. The conviction and sentence are vacated.
CLASS NOTES
Understand the Abandonment (Renunciation) Principle
o A person is not guilty of attempt if: (1) she abandons her effort to commit the
crime or prevents it from being committed; and (2) her conduct manifests a
complete and voluntary renunciation of her criminal purpose.
HYPO- Bank of America
It seems like she abandoned her criminal purpose, BUT there is not a complete and
voluntary renunciation of her criminal intent because she left with the mindset that she
couldnt complete this crime alone because the bank was heavily guarded and she needed
help from her cousin. Therefore she did not fulfill the requirements.
o If someone decides they wont do something right then because they assessed the
situation, and decided to try again later. (cameras, security) NOT
ABANDONMENT
o If someone has a complete change of heart and stops because they truly dont
want to do it anymore and have no intention of attempting it ever again, they DO
SATISFY ABANDONMENT.

C. Solicitation
Solicitation-The criminal offense of urging, inciting, or counseling someone to commit
an unlawful act.

-IMPORTANT!! No solicitation occurs if the solicitor attempts to commit the act


himself OR if he asks for assistance from another party.
State v. Mann Inchoate Offenses Solicitation Specific Intent Crime
Solicitation is a controversial crime because the offense is complete once the
solicitor asks, entices, or encourages another to carry it out.
The solicitor is considered more dangerous than the solicitee because the solicitor
instigates the crime.
The solicitor is also considered more morally culpable, since he seeks to shield
himself from liability by soliciting another to carry out the actual crime.
The offense of solicitation merges into the crime solicited if the latter offense is
committed or attempted by the solicited party.
o Examples in casebook.
CLASS NOTES
33

Inchoate Offenses Solicitation- Specific Intent Crime


o Solicitation involves asking, enticing, inducing, or counseling of another to

commit a crime.
o A solicitor
Plans, schemes, suggests, encourages, and incites
Offense is complete as soon as the solicitor asks, entices, or encourages another to
commit the target offense. (Completion of the crime isnt necessary)
What is Merger
o (Conspiracy, attempt, or completed crime)
What is the Innocent Instrumentality Rule?
o Someone thats completely unaware that they are engaging in criminal conduct.

HYPO (Made up) Innocent Instrumentality Rule


Kelseys sitting on her porch and Momo comes up and starts a convo. He then tells her
that he broke his arm and was wondering if she could go across the street and get his bike
because hes too weak to push it. She does as he asks and walks his bike back to her
porch. Unbeknownst to her, it is not his bike and she just stole someone elses bike
believing that it was his. Would Kelsey be guilty of solicitation?

No, because Kelsey didnt know that she was doing anything
unlawful. Therefore, Momo didnt solicit a crime because Kelsey
was unaware that any crime was being committed.
State v. Cotton- Cotton (D) was charged with the crime of engaging in sexual conduct with his
stepdaughter. While in prison, Cotton wrote two letters to his wife, telling her to convince his
stepdaughter not to testify against him. He instructed his wife to offer his stepdaughter money to
leave the state and to tell her that testifying wouldnt be nice and would likely place her in the
news. Unknown to Cotton, his cellmate intercepted the first letter and turned it over to the
authorities when D asked him to get a stamp for him to mail it. He never mailed the second letter.
Cotton was charged with two counts of criminal solicitation and convicted.
Rule- An uncommunicated solicitation is insufficient to constitute the offense of criminal
solicitation.
Holding- The lack of actual communication prevents Cottons conviction under the
solicitation statute. Therefore, Cottons convictions for solicitation are reversed.
Proof that a defendant meant to communicate a solicitation is insufficient to constitute the
offense of criminal solicitation.
o There must be proof that the solicitation was actually communicated to the
solicitee.
This is distinct from the Model Penal Codes approach. The Model Penal Code only
requires that the defendant intended to communicate the solicitation. **NOT TESTED*
When this jurisdictions Legislature adopted the Model Penal Codes definition of
solicitation in its criminal code, it specifically omitted the provision on
uncommunicated solicitations.

34

o This suggests that the Legislature meant to require actual communication from the
defendant to the solicitee.
The lack of actual communication prevents Cottons conviction under the solicitation
statute. Cotton is at most guilty of attempted solicitation.

CLASS NOTES
Inchoate Offenses - Solicitation
o Solicitation requires some form of actual communication from Defendant
o The communication can be to either an intermediary or the person intended
to be solicited.
Does solicitation require actual communication
ONLY COMMON LAW TESTED
HYPO- The Punch
Should he be convicted for solicitation?
No, because its a Specific Intent Crime. He said it in jest (jokingly) so
therefore didnt have the specific intent to commit the crime and take the
punishment. He didnt mean for him to do that.
HYPO- The Pocket Problem
Which is the most accurate answer?
A- NO. Momo prevails because of legal impossibility?
B- NO. Momo prevails because it was factually impossible for him to p/p check.
C- NO. Momo will be convicted because Chris attempted to pickpocket Ralph.
D- Momo will be convicted because of the Mens Rea component of the crime.
HYPO- Sniper
C- NO, Ralphie should not be convicted of solicitation, because he was the
solicitor who was asking Momo to help him with a crime that he was going to take
the lead on. Therefore, the requirement for solicitation isnt satisfied because
Ralphie was going to do it himself, AND he asked Momo for assistance.
Ralphie asked Momo for help, but not to do it himself. Therefore, he
shouldnt be charged.
**Important!! No solicitation occurs if the solicitor attempts to commit the
act himself OR if he asks for assistance from another party.
D. CONSPIRACY
Conspiracy- An agreement between two or more individuals to commit an illegal
act at some point in the future and often requiring at least one overt act in
furtherance of that agreement.

35

People v. Carter

A criminal conspiracy exists when two or more people agree to commit an unlawful act.
The crime of conspiracy does not require an overt act.
Thus, as soon as the partnership agreement is made, the crime of conspiracy has been
committed.
The crime of conspiracy does require evidence of 2 specific purposes.
o First, the conspirator must intend to partner with others.
o Second, the conspirator must intend to commit the crime.
Significantly, the crime of conspiracy does not merge into the completed crime.
Thus, a conspirator can be convicted & charged with conspiracy & the completed crime.

CLASS NOTES
Inchoate Offenses Conspiracy
o A partnership in criminal purposes
o A mutual agreement or understanding between two or more persons to commit a
criminal act

Pinkerton v. United States- Walter and Daniel Pinkerton (Ds) were brothers who were
indicted for various violations of the Internal Revenue Code (IRC). A jury found Walter guilty on
nine of the ten substantive counts and on a conspiracy count, and found Daniel guilty on six of
the ten substantive counts and on the conspiracy count. Each brother was fined and sentenced to
terms of imprisonment. Daniel and Walter appealed, and their convictions were affirmed by the
circuit court of appeals. The U.S. Supreme Court granted certiorari to review.
Rule- In an ongoing conspiracy, the overt (clear) act of one co-conspirator may be the act
of all conspirators without any new agreement specifically directed to that act.
Holding
There is no evidence that Daniel directly participated in the commission of the
substantive offenses.
o However, the evidence showed the illegal acts committed by Walter were done in
furtherance of an unlawful agreement, i.e., conspiracy, between Walter and
Daniel.
Daniel argues that his alleged participation in a conspiracy is not enough to sustain his
conviction on the substantive offenses even though Walter committed them in furtherance
of the conspiracy.
Under United States v. Sall,- When two or more individuals conspire to commit illegal
acts, the criminal intent to commit the substantive offenses has been achieved, regardless
of who actually commits the illegal substantive offenses.
o Further, the conspiracy is ongoing until one of the co-conspirators affirmatively
withdraws from it.
Therefore, the ongoing conspiracy is attributable to all co-conspirators
until one intentionally removes himself from it.
There are some exceptions:

36

o If its not done in furtherance of the crime


o If theres a clear withdrawal
o Not a necessarily foreseeable consequence
Here, the acts were done in execution of the conspiracy.
An overt act is an essential ingredient of the crime of conspiracy. If one conspirator can
supply that act, there is no reason why other acts in furtherance of the conspiracy cannot
be equally attributable to other members.
The judgments of conviction are affirmed.

CLASS NOTES

People v. Lauria- A police investigation revealed that three known prostitutes were using
Laurias (D) telephone answering service for business purposes. Stella Weeks, a police officer,
went undercover, posed as a prostitute, and signed up with Laurias answering service. Over
approximately a three month period, Weeks periodically complained to Laurias office manager
about losing calls and not receiving messages for tricks. Lauria defended his service and
emphasized, his business was taking messages. Thereafter, Lauria and the three prostitutes
were arrested and charged with conspiracy to commit prostitution, a misdemeanor. Lauria
objected to the arrest and told the police that, while he knew of only one known prostitute, his
records were always available whenever the police had a specific name to investigate, but also
that his service did not arbitrarily tell the police about prostitutes. The trial court dismissed the
indictment brought against Lauria and the three prostitutes as lacking probable cause. The People
appealed.
Rule- In order to make a supplier of goods or services a participant in a criminal
conspiracy, the prosecution must show the supplier had knowledge of the illegal use of
the goods or services and had the intent to further the illegal use of the goods or services.
Holding- The order dismissing the indictment is affirmed.
The prosecution attempted to establish the element of an agreement to further an illegal
act, required to show the presence of a conspiracy, by showing that Lauria knew his codefendants were prostitutes who used his service to receive business calls and
continued to furnish them with the service.
o This approach attempts to equate knowledge of anothers criminal activity with a
conspiracy to further that illegal activity. In United States v. Falcone where sellers
of sugar, yeast and cans, who had only knowledge of an illegal liquor conspiracy,
were held not to have participated in the conspiracy with distillers who purchased
the goods from them.
o In Direct Sales Co. v. United States, a wholesaler of drugs was convicted of
conspiracy to violate the federal narcotic laws by selling drugs in quantity to a codefendant physician who was supplying them to addicts.
There, the wholesaler actively promoted the sale of morphine sulfate and
sold the drug in quantities 300 times his normal requirements.
The Court in Direct Sales said that there must be more than just
knowledge of illegal activity to be a conspiracy; there must be intent to
further it and a stake in the venture.

37

To show intent, it is easiest if there is proof of direct participation in the illegal acts.
o But when such evidence is lacking, intent may be inferred when the supplier of
services has a stake in the illegal venture.
o Here, there is no proof that Lauria took any direct action to further, encourage, or
direct the prostitution activities of his co-defendants, and the order dismissing
the indictment is affirmed.

Iannelli v. United States- Iannelli (D) was convicted of conspiring to violate 18 U.S.C.
1955, which criminalized the joining together of five or more people to run an illegal gambling
business. Iannelli was convicted of both the conspiracy and the targeted offense. The Court of
Appeals for the Third Circuit affirmed.
Rule- Whartons Rule does not preclude a conviction of criminal conspiracy where such
a result is contrary to legislative intent.
Whartons Rule- A rule that prevents a conviction of conspiracy to commit a crime that
only two or more people can commit.
o Example- 2 people cant be found guilty of adultery since it takes two people to
engage in adultery.
Holding
Ordinarily, the crime of conspiracy and its targeted offense do not merge, and both may
be punished as distinct crimes.
A major exception to this principle is Whartons Rule, which states that a conspiracy
should merge into the targeted offense if only two or more people can commit the
targeted offense.
o Some classic examples of Whartons Rule offenses include adultery, incest, and
dueling.
Corrupt Motive Rule- you must have a corrupt motive to commit a crime within the
agreement within your partnership.
But Whartons Rule is merely a judicial presumption that is overcome when logic and
legislative intent are to the contrary.
o For instance, the rationales underlying the Wharton Rule do not apply to the crime
at issue here. Section 1955 criminalizes the combination of five or more people to
run a gambling business prohibited by state law.
Unlike classic Wharton Rule offenses that only involve and affect the conspirators,
running an illegal gambling business tends to involve and affect both conspirators and
non-conspirators, such as bettors
**It is likely that the Legislature intended to prevent the formation of large-scale
gambling businesses by criminalizing the conspiracy to form them.
o Because the Legislature has made known a purpose that runs contrary to
Whartons Rule, the presumption is destroyed.
This Court must assume that the Legislature intends that a conspiracy to violate 1955
be treated as an offense distinct from the targeted offense.

CHAPTER 11- Liability For The Conduct of Another

38

A. Accomplice Liability
1. Common Law Terminologies and its Significance
State v. WardUnder common law, Criminal Accomplices can be divided into 4 categories:
o Principals in the first degree
A principal in the first degree is one who perpetrates the crime.
o Principals in the second degree
A principal in the second degree is one who is present during the
commission of the crime and actively encourages the commission of the
crime.
A principal in the second degree may be convicted even if the
principal in the first degree has not yet been tried or has been
acquitted OR convicted of a more serious crime than P in 1st.
o Accessories before the fact
An accessory before the fact is one who is not present during the
commission of the crime but provides assistance.
o Accessories after the fact
An accessory after the fact is one who helps a felon evade detection after
the commission of the crime.
A principal in the second degree may be convicted even if the principal in the first degree
has not yet been tried or has been acquitted.
o Notably, a principal in the second degree may be convicted of a more serious
crime than a principal in the first degree.
In contrast, an accessory cannot be tried before the principal without his consent, and
cannot be convicted of a more serious crime than the principal.
Principal- A person who appoints another person to act as his or her agent.
Accessory- An accomplice who is not present during the commission of the crime but
aids or encourages the commission of the crime.
2. Elements of Accomplice Liability: In General

State v. Hoselton- Hoselton (D) was charged with entering without breaking, with the intent to
commit larceny. Hoselton had been standing on a barge while several of his friends entered a
storage unit at the other end of the barge. Hoselton was unaware that his friends intended to steal
items from the storage unit until he walked closer and saw his friends removing the goods
Hoselton neither helped his friends place the items in the car, nor received any of the items. At
trial, the State (P) asked Hoselton whether he was a lookout for his friends. Hoselton replied,
You could say that, and stated that he just did not want to be present during the commission of
the crime. Hoselton was convicted as a principal in the first degree. Hoselton appeals, arguing
that there is insufficient evidence to support his conviction.
Rule- A defendant is not a principal in the second degree unless he participates in the
crime while sharing the criminal intent of the principal in the first degree.
A lookout is one who participates in a crime by keeping watch, in order to ensure that the
criminal acts of his accomplices are undetected.
39

o A defendant who participates in a crime as a lookout may be convicted as a


principal in the second degree.
Principals in the second degree are punishable to the same extent as principals in the first
degree.
However, in order to be convicted as a principal in the second degree, the prosecution
must establish that the defendant participated in the crime while sharing the same
criminal intent as the principal in the first degree.
Here, the State attempts to demonstrate that Hoselton was a lookout for his friends, and
should therefore be convicted of the crime as a principal in the second degree.
o However, the State failed to establish that Hoselton participated in the offense
with the same criminal intent as his friends.
Hoselton had no knowledge of his friends intentions until they began stealing the items.
When he realized what they were doing, he left the scene and went to their car.
He neither handled the goods nor received them.
Furthermore, Hoseltons vague testimony as to whether he was a lookout for his friends
does not demonstrate that he shared his friends intent to commit larceny.
Accordingly, the conviction for entering without breaking is reversed.

4. Actus Reus
State v. V.T.- V.T. (D) and two friends stayed the night at the home of one of his relatives.
Two days after they left, the relative discovered that her camcorder was missing. She
reported the theft to the police. Police found the camcorder at a pawn shop. Inside the
camcorder was a tape that showed V.T. and his two friends. The footage shows one of
V.T.s friends on the phone discussing his plans to pawn the camcorder. V.T. is shown in
the footage, as well. He neither speaks nor gestures during the phone call. V.T. was
charged with the theft of the camcorder. The juvenile court found him guilty.

Rule- Evidence of mere presence is insufficient to demonstrate complicity in a crime.


Accomplice liability may be imposed where a defendant solicits, commands,
encourages, or assists in the commission of a crime.
o But there must be evidence that the defendant affirmatively participated in the
commission of the crime.
Passive behavior, such as mere presence at the scene of the crime, is insufficient to
impose accomplice liability.
There is no evidence that V.T. took any affirmative action to solicit, command,
encourage, or assist in the theft of the camcorder.
Because the evidence was insufficient to demonstrate V.T. was an accomplice, the
juvenile courts finding of guilt was in error.

CHAPTER 12- Theft


A. Larceny
40

1. Actus Reus
A. Tresspassory Taking (Caption) and Carrying Away (Asportation)
U.S. v. Mafnas- Mafnas (D) was an employee of Guam Armored Car Service. He transported
bags of money belonging to the Bank of Hawaii and the Bank of America. On three separate
occasions, Mafnas took money from the bags. He was tried and convicted under a federal statute
that prohibits taking with the intent to steal money from a bank.
Rule- One who receives property for a limited or temporary purpose does not have lawful
possession of the property.
Holding: Federal law prohibits one from taking with the intent to steal money from a
bank.
This court has construed this provision to adopt the requirements of common law larceny.
Therefore, a taking under this provision must be trespassory.
Where a defendant has lawful possession of property, the defendants misappropriation of
that property is not trespassory.
Thus, Mafnas taking of the money was trespassory, whether Mafnas was a bailee or not,
and the conviction is affirmed.
B. Embezzlement
Rex v. Bazeley- Bazeley (D) was a teller at a bank owned by Esdaile and Hammett. In January
1799, a customer deposited bank notes and cash for an account. Bazeley deposited most of what
he received into the account, but placed one bank note in his pocket, which he later converted to
his own use. He was prosecuted for theft.
Rule- Where a defendant employee has been entrusted with the employers property, the
employees misappropriation of the property does not constitute larceny.
Embezzlement- The fraudulent conversion of another's property by a person who is in a
position of trust, such as an agent or employee.
Holding- In order for an employee to commit the crime of larceny, the employee must
take the property from the possession of the employer.
This means the employer must have actual or constructive possession over the property at
the time the employee misappropriates it.
An employer is only in possession of the property if he has the right to and control over
the property.
Where an employer does not have possession of the property, the employees conversion
of the property is not larceny, but merely a breach of trust.
This type of breach of trust is not a felony under common law or statute.

C. False Pretenses
People v. Whight- Whight (D) opened a checking account, for which he received an ATM card.
41

Within a few months, he had overdrawn on the account. After Whight failed to replenish his
account within a certain timeframe, the bank closed his account. But Whight afterwards
discovered that the ATM card still worked at certain Safeway markets. Safeway allowed market
customers to get cash back when they made purchases with an ATM card. Each time a customer
used an ATM card, Safeways computer system communicated with Wells Fargo to verify the
cards. Wells Fargo would contact the customers banks and would issue a code of approval or
disapproval to Safeway. But if Wells Fargo was unable to communicate with the customers
bank, it would simply issue Safeway a stand-in code to approve the transaction. Safeway would
issue the money and later verify the ATM card. After discovering that he could obtain money
through this system, Whight used his ATM card at four Safeway markets and withdrew over
$19,000. Each time, the computer system had an error that caused Wells Fargo to issue Safeway
a stand-in code. Whight was subsequently convicted for grand theft by false pretenses.
Rule- An owner is deemed to pass title to a defendant in reliance upon the defendants
misrepresentation even where the owner has undertaken an investigation of the
defendants claim.
Grand Theft- An owner is deemed to pass title to a defendant in reliance upon the
defendants misrepresentation even where the owner has undertaken an investigation of
the defendants claim.
Holding The crime of false pretenses has THREE elements.
o First- the defendant must make a misrepresentation.
o Second- the misrepresentation must be made with the intent to defraud the owner
of his property.
o Third- the owner must actually be defrauded of his property due to the
misrepresentation.
It is essential that the owner pass title to the defendant in reliance upon the defendants
misrepresentation.
If the misrepresentation had a material influence on the owner, false pretenses can be
proven.
What is more, even if an owner personally investigates the defendants representations
before passing title, this does not mean the owner did not rely upon the defendants
misrepresentation. As long as the owner does not rely solely upon his own investigation,
a conviction of false pretenses can be sustained.
Here, Whight falsely represented to the Safeway clerks that his ATM card was valid.
Whight argues that Safeway did not act in reliance upon Whights misrepresentation
because it instead relied upon Wells Fargos verification.
But Safeway did not rely upon Wells Fargo. In fact, Wells Fargo did not inform Safeway
whether or not the ATM card was valid. It merely issued a stand-in code, demonstrating
that the validity of Whights card was unverified.
Upon receiving the stand-in code, Safeway was forced to rely solely upon Whights
representation that the card was valid.
Therefore, the element of reliance is present in this case.

42

QUIZ 3
Week of April 18th
8-10 Questions- Comprehensive
FOCUS ON DETAILS
Cumulative- Focus on- Specific & General Intent, Attempt, Solicitation
Question similar to Sniper Hypo

43

**MIDTERM QUESTIONS**
1.) Sal is very intoxicated and moves towards Ralphie. Ralphie claims self-defense in Sals
death
C- Ralphie will likely not prevail because he knew, or shouldve known that he could
have easily disarmed Sal again. 18.01 C in the supplement. Necessity Component.

2.) Tony is walking down the street with this GF. Don calls Simone a disgrace. Tony gets
mad and pulls out a knife and threatens him. Who is the AGGRESSOR?
C- Even though Tony didnt use the knife, he was the aggressor due to his weapon.
o Define Aggressor3.) Ex post Facto= Legislature problems
4.) Momo wants to rob a store so he shoots two shots in the air, Simi has a heart attack during the
robbery, but he had headphones on the whole time and was totally unaware of the robbery in
action. Is Momo liable for felony murder in relation to Simis death?
D- No casual relationship between the robbery and the death, so no felony murder.
5.) Archie likes to sell scrap metal. He goes into a Federally owned dumping ground. Which
answer best describes Archies Circumstance?
A- Archie is not guilty, provided he reasonably believed the bomb casings he took where
abandoned. General Intent Crime.
REFERENCE- (U.S. v. Veach or 158-159 Specific and General Intent.)
6.) Koresh belongs to a religious cult and he believes that the plane hes ordered to crash will go
down but only killing his wife and not the other 10 people because he prayed about the rest of
them. Which answer is most accurate regarding his first-degree murder charge?
B- He is not guilty because he was not aware that the 10 other passengers would die.
7.) Frank and his brother robbed a liquor store and fired warning shots. Shop owner tried to stop
them by shooting but accidentally shot a customer.
D- Frank is not guilty.
8.) Shortly after giving birth to Sarah, Dawn went on a drug bings where she forgot to feed,
bathe, and clothe Sarah during that time and she died 53 days after birth. Should she be
convicted of Felony Murder Rule under the Abstract Test?
A- No because the
9.) Momo stabs Rob and the doctor negligently performs a surgery, and Rob dies. Who is
responsible for his death?
C- Momo because his original conduct lead to the death of Rob.

44

10.) Walking down Michigan Ave. late one night, Rob sees a man take a girl to the ground
intending to get her to lay face down. After she yells rape, Rob pushes the man off of her and
tells her to run. He later finds out he was an FBI lawfully arresting her. Prosecutors charge him
for assaulting an officer and he claims defense under the alter ego rule
B-Because Zena had no legal right to resist.
11.) B lives in Englewood a high crime area. He notices that the police never come when he calls
or if they do they come hours later. On their walk home his sisters life is threatened and since the
police never help, B turns to the local gangster for help and he helps him but later forces him to
kill Emma. HE is charged with murder and claims he acted under duress.
D- B should be found guilty because Duress isnt a defense in murder.
12.) B invited some friends over to binge watch a tv show and drink. He later found Bebo in bed
with his wife so he charged at him with a knife and
D- Bebo can claim self-defense
13.) Momo robbed a liquor store and a cop chases after him and falls through air-shaft and dies.
Momo is charged with Felon Murder.
C- Prosecution because he dies in pursuit of a felony pursuing felonious conduct.
14.) Alice planned to kill Barb but she came home to find barb in bed with her husband
D- Alice should be found guilty of murder
15.) D is told he has 2 years to live and the only way to save his life is to illegally smoke pot. He
claims Necessity as his defense. Which best described the outcome?
B- Santos wont prevail because necessity is not a proper defense for legislature
16.) . A person who voluntarily violates this crime will be charged. Which is most
accurate if the jury believes Camille?
A- Camille should be acquitted.
17.) Joe visits his ex wife to discuss their children. She unknowingly to Joe, fills his drink with
tasteless vodka and is arrested for a DUI. They live in a strict liability jurisdiction and Joe pleads
innocent to the DUI. He tries to claim voluntary intoxication, will he prevail?
C- Joe may not claim voluntary intoxication as a defense.
18.) Donald threatens to kill Maggie once she has her baby in 3 months and when he turns to
walk away she stabs and kills him. If she tries to use the self-defense claim, will she prevail?
No because the threat was not imminent.

45

You might also like